« Trigonométrie/Exercices/Résolution de systèmes » : différence entre les versions

Contenu supprimé Contenu ajouté
m Robot : Remplacement de texte automatisé (-c.-à-d. +c'est-à-dire)
 
Ligne 64 :
:<math>\begin{cases} \sin(x+y)=\cos(x-y) \\ \tan x-\tan y=1.\end{cases}</math>
{{Solution|contenu=
Ce système équivaut à <math>\left(1-\tan x\right)\left(1-\tan y\right)=0\text{ et }\tan x-\tan y=1</math>, c.'est-à-d.dire à <math>\left(\tan x=1\text{ et }\tan y=0\right)\text{ ou }\left(\tan y=1\text{ et }\tan x=2\right)</math>. Ses solutions <math>\pmod\pi</math> sont donc <math>\left(\frac\pi4,0\right)</math> et <math>\left(\arctan2,\frac\pi4\right)</math>.
}}
 
Ligne 102 :
'''3°''' &nbsp;<math>\begin{cases}\cos2x+\cos2y=1\\\cos x+\cos y=m.\end{cases}</math>
{{Solution|contenu=
'''1°''' &nbsp;Si <math>a=b=0</math>, les solutions <math>\{x,y\}</math> sont <math>\{x,x+(2k+1)\pi\}\quad x\in\R,k\in\Z</math>. Supposons maintenant <math>a^2+b^2\ne0</math>. Alors, <math>y\mod2\pi</math> sera entièrement déterminé en fonction de <math>x</math> par son cosinus {{nobr|(<math>a-\sin x</math>)}} et son sinus (<math>b-\cos x</math>), sous réserve que ces deux données soient compatibles, c.'est-à-d.dire que <math>(a-\sin x)^2+(b-\cos x)^2=1</math>, ce qui équivaut à
::<math>\frac b{\sqrt{a^2+b^2}}\cos x+\frac a{\sqrt{a^2+b^2}}\sin x=\frac{\sqrt{a^2+b^2}}2</math>.
:On détermine les solutions <math>x</math> par la méthode habituelle, cf. [[../../Équations et inéquations trigonométriques#Cas général|Cas général]] (il en existe si et seulement si <math>a^2+b^2\le4</math>).
'''2°''' &nbsp;Il faut bien sûr supposer <math>a\ne0</math>. Le système équivaut alors à <math>\sin(x+y)=\cos x\cos y=a</math>.
:Si <math>|a|>1</math>, pas de solution. Si <math>|a|\le1</math>, <math>x+y\mod2\pi</math> est déterminé par <math>\sin(x+y)=a</math> et <math>\cos(x+y)=\varepsilon\sqrt{1-a^2}</math>, avec <math>\varepsilon=\pm1</math>, et la seconde équation équivaut alors à <math>\cos(x-y)=2a-\varepsilon\sqrt{1-a^2}</math>. Or <math>\left(2a-\varepsilon\sqrt{1-a^2}\right)^2\le1\Leftrightarrow\varepsilon a=|a|\le\frac45</math>.
:Finalement, il y a des solutions si et seulement si <math>|a|\le\frac45</math>, et elles sont alors données, pour <math>\alpha,\beta\in\R</math> tels que <math>\sin\alpha=a</math>, <math>\cos\alpha=\frac a{|a|}\sqrt{1-a^2}</math> et <math>\cos\beta=2a-\frac a{|a|}\sqrt{1-a^2}</math>, par : <math>x+y\equiv\alpha\mod2\pi</math> et <math>x-y=\pm\beta\mod2\pi</math>, c.'est-à-d.dire <math>y\equiv\alpha-x\mod2\pi</math> et <math>x\equiv\frac{\alpha\pm\beta}2\mod\pi</math>.
'''3°''' &nbsp;Le système équivaut à <math>\cos x=u,\cos y=v</math> (ce qui détermine <math>x,y</math> à <math>\pm</math> près et <math>\mod2\pi</math>), avec <math>u+v=m\text{ et }uv=P:=\frac{m^2}2-\frac34</math>.
:Cf. [[../../Équations et inéquations trigonométriques#Condition d'existence de deux cosinus (ou deux sinus) de somme et produit prescrits|Condition d'existence de deux cosinus de somme et produit prescrits]] : il existe des solutions si et seulement si <math>|m|-\frac14\le\frac{m^2}2</math> et <math>|m|\le\sqrt3</math>.
Ligne 124 :
{{Solution|titre=Solution de la question 2, particulièrement difficile|contenu=
'''2°''' &nbsp;Les solutions sont déterminées <math>\mod2\pi</math> par leur cosinus et le signe de leur sinus.
:Le signe de <math>a</math> détermine celui de <math>\sin x\sin y</math> et outre cette contrainte, le reste du système équivaut, en notant <math>u=\cos x</math> et <math>v=\cos y</math> et <math>P=uv</math>, à <math>\left(1-u^2\right)\left(1-v^2\right)=a^2\text{ et }u+v=b</math>, ou encore : <math>P^2+2P+1-b^2-a^2=0\text{ et }u+v=b</math>, c.'est-à-d.dire <math>P=-1\pm\sqrt{a^2+b^2}\text{ et }u+v=b</math>. Les 2 solutions <math>u,v</math> de <math>z^2-bz+P=0</math> appartiennent à <math>\left[-1,1\right]</math> si et seulement si <math>b^2\ge4P</math>, <math>|b|\le2</math> et <math>|b|\le P+1</math>. Il faut donc que <math>P=-1+\sqrt{a^2+b^2}</math> (et non pas <math>-1-\sqrt{a^2+b^2}</math>), et la condition d'existence de solutions <math>\left(u,v\right)\in\left[-1,1\right]^2</math> est alors : <math>4\sqrt{a^2+b^2}\le b^2+4\le8</math>.
}}